Writing A Map As A Composition of Three Linear/Bilinear Maps

Click For Summary
The discussion revolves around expressing the map F: C0([a,b], Rn) --> R, defined as F(φ) = ∫φ(t)φ(t)dt, as a composition of three linear or bilinear maps. Participants are exploring potential mappings, suggesting that F could be one of the maps, and discussing the nature of the required intermediate maps. There is uncertainty about how to represent the product φ(t)φ(t) and whether a more general mapping is needed. The conversation emphasizes the need for clarity in identifying the correct composition and applying the chain rule and generalized product rule for differentiation. Overall, the thread highlights the complexities involved in decomposing the integral map into simpler components.
azdang
Messages
80
Reaction score
0

Homework Statement


Consider the map F: C0([a,b],Rn) --> R, F(\phi)=\int\phi(t)\phi(t)dt (This is the integral from a to b). Write F as a composition of three maps, each of which is linear or bilinear. Then use the chain rule and generalized product rule to compute DF(\phi).



Homework Equations





The Attempt at a Solution


The only maps I could think of would be something like this:

\phi: [a,b] --> Rn

A: C0([a,b],Rn) --> C0([a,b],Rn)

And then maybe I would need a map for \phi\phi? But I'm not sure what that would be or if that's even correct. I'm not even sure that I am on the right track at all with the maps. Does anyone have any ideas?

Is it possible that F is one of the three maps? So that the composition becomes F of A of phi?
 
Physics news on Phys.org
Of course, \int f(t)dt is a linear function, from the set of integrable functions to the set of real numbers, itself. Are you including that? And f(x,y)= xy is multi-linear.
 
I'm not sure I know what you mean.:confused:

Isn't F already that map of integrable functions? Or will I need a more general one such as I: C0([a,b],Rn) --> Rn as one of my maps?

Then, what I'm thinking is that it should be:

I of 'something' of phi. I can't figure out what that middle map would be though.
 
Last edited:
Question: A clock's minute hand has length 4 and its hour hand has length 3. What is the distance between the tips at the moment when it is increasing most rapidly?(Putnam Exam Question) Answer: Making assumption that both the hands moves at constant angular velocities, the answer is ## \sqrt{7} .## But don't you think this assumption is somewhat doubtful and wrong?

Similar threads

  • · Replies 3 ·
Replies
3
Views
2K
  • · Replies 6 ·
Replies
6
Views
2K
Replies
9
Views
2K
  • · Replies 2 ·
Replies
2
Views
2K
Replies
3
Views
2K
Replies
1
Views
1K
  • · Replies 36 ·
2
Replies
36
Views
5K
  • · Replies 4 ·
Replies
4
Views
2K
  • · Replies 9 ·
Replies
9
Views
2K
  • · Replies 2 ·
Replies
2
Views
3K